Solving Integral Equations: Placing Arbitrary Constants in ln/e Solutions

  • Thread starter Thread starter Air
  • Start date Start date
  • Tags Tags
    Integration
Click For Summary
SUMMARY

The discussion centers on solving the differential equation y' = 3y, with a focus on the placement of arbitrary constants in solutions involving natural logarithms and exponentials. The user initially derives the solution as y = e^{3x} + e^{3c}, while the textbook states the correct form is y = ce^{3x}. The key takeaway is the importance of correctly applying the properties of exponents, specifically that e^{x+y} equals e^x * e^y, not e^x + e^y.

PREREQUISITES
  • Understanding of differential equations, specifically first-order linear equations.
  • Familiarity with integration techniques involving natural logarithms.
  • Knowledge of properties of exponents and logarithms.
  • Basic algebraic manipulation skills.
NEXT STEPS
  • Study the method of solving first-order linear differential equations.
  • Learn about the properties of logarithmic and exponential functions.
  • Explore the concept of arbitrary constants in differential equations.
  • Practice problems involving integration of exponential functions.
USEFUL FOR

Students and educators in mathematics, particularly those focusing on calculus and differential equations, as well as anyone seeking to clarify the application of arbitrary constants in integral solutions.

Air
Messages
202
Reaction score
0
This is a question...

For the following question:
y^{'}=\frac{dy}{dx}=3y

I get the solution...
\int \frac{1}{3y} dy = \int dx
\frac{1}{3}ln y = x + c
y = e^{3x}+e^{3c}

However the textbook example says the solution is...
y = ce^{3x}

My question is would my answer be incorrect? How should the arbitrary constant be placed in ln and e integral solutions?
 
Physics news on Phys.org
e^{x+y}=e^xe^y and not e^{x+y}=e^x+e^y. You made this mistake in your last line.
 
You have to take into account that
e^{x+y}=e^x \cdot e^y \neq e^x + e^y
To get a feeling for that relation, take for example
2^{3+4}=(2 \cdot 2 \cdot 2 )\cdot (2 \cdot 2 \cdot 2 \cdot 2 )=2^3 \cdot 2^4
 
Oh yes, that is correct. Silly mistake. Thanks.
 
Question: A clock's minute hand has length 4 and its hour hand has length 3. What is the distance between the tips at the moment when it is increasing most rapidly?(Putnam Exam Question) Answer: Making assumption that both the hands moves at constant angular velocities, the answer is ## \sqrt{7} .## But don't you think this assumption is somewhat doubtful and wrong?

Similar threads

  • · Replies 4 ·
Replies
4
Views
1K
  • · Replies 14 ·
Replies
14
Views
2K
  • · Replies 3 ·
Replies
3
Views
1K
  • · Replies 25 ·
Replies
25
Views
2K
  • · Replies 6 ·
Replies
6
Views
2K
  • · Replies 2 ·
Replies
2
Views
1K
Replies
4
Views
2K
  • · Replies 8 ·
Replies
8
Views
2K
Replies
3
Views
2K
  • · Replies 12 ·
Replies
12
Views
3K